9

Suppose $A$ and $B$ are operators, $A$ is Hermitian, $B$ anti-hermitian, and their commutator is the identity, i.e. $$[A, B] = I \, .$$ Denoting the eigenvectors of $A$ as $\lvert a \rangle$, so that $A \lvert a \rangle = a \lvert a \rangle $, we have $$\langle a| [A, B] |a \rangle = \langle a | a \rangle \tag{1}$$ and $$ \langle a| (AB-BA) |a \rangle = a \langle a | B | a \rangle - a \langle a | B | a \rangle = 0 \, . \tag{2}$$

Summarizing, if $|a\rangle$ is normalized, $\langle a|a\rangle = 1$, we obtain $0=1$. Where is the mistake?

Nikodem
  • 676

2 Answers2

10

This is a very nice problem in the theory of operators in separable Hilbert spaces. The trick to notice is that your $|a\rangle$ is not in the domain of the commutator, therefore your equation 1) is meaningless. More precisely, we have the following lemma:

Lemma Let C be the commutator C(A, B; D(C)), in the sense that: $$\forall \phi \in D(C) \subseteq D(AB) \cap D(BA), (AB-BA)\phi = C\phi$$ Suppose furthermore that A is selfadjoint with a non-empty point spectrum. A necessary condition for the eigenvectors of A to belong to D(C) is that C maps each of the eigenvectors of A to its orthogonal complement.

Proof . Let Aϕ = aϕ and ϕ ∈ D(C). Because the eigenvalue a is real and because $|\langle ϕ|Bϕ\rangle| < ∞$, we have the equality $\langle ϕ|Cϕ\rangle = \langle ϕ|(AB − BA)ϕ\rangle = 0$. That is $\langle ϕ|Cϕ\rangle = 0,$ or ϕ is orthogonal to Cϕ.

Assume now that $C = 1_{\mathcal{H}}$. Then, in order to have the eigenvector $\phi$ in the domain of the commutator, it follows by the previous lemma that $\phi = 0$.

DanielC
  • 4,333
  • Beautiful answer to a very important and popular question among theoretical physicists! The crucial point is " |a⟩ is not in the domain of the commutator". Thanks! – Nikodem Nov 13 '22 at 01:19
4

@Qmechanic answered your question in the comments, but evidently the message did not automatically sink in. Let me try to illustrate it with the routine demonstration you probably were exposed to when learning the uses of the Dirac bra-ket notation.

The short answer is that the tracefulness of the identity in the r.h.s. of your commutator equation for an infinite dimensional Hilbert space leads to $\langle a | a\rangle \neq$1, because it is actually singular. So your equation 1) is fine, since the r.h.s. is infinity. But your equation 2) is flawed, since the relevant expression involves a 0 multiplying a stronger infinity, amounting to infinity, again, as in 1).

I will illustrate this with $A=\hat x$ and $B=\hat p /\hbar$, as in standard QM courses. Absorb $\hbar$ in $\hat p$ to make the formalism more familiar.

Starting from the standard operator equation $[\hat x ,\hat p]=i 1\!\!1 $, first take its non-diagonal matrix elements, before building up to your 2), $$ \langle x|\hat x \hat p - \hat p \hat x|y\rangle = (x-y)\langle x|\hat p|y\rangle=(x-y)\int dp ~ \langle x| p\rangle \langle p| \hat p|y\rangle \\ =(x-y)\int dp ~ \langle x| p\rangle p \langle p |y\rangle \\ =\frac{ (x-y)}{2\pi}\int dp ~ p ~e^{i(x-y)p} =-i (x-y)\partial_x \delta (x-y) \\ =i \delta(x-y). $$ As always, $\langle x| p\rangle=\exp(ixp) ~/\sqrt{2\pi}$.   Check the last equality by operating on a well-behaved test function. It trivially reflects homogeneity of degree -1, $\delta(\lambda x)=\delta(x)/\lambda$, so differentiate this by λ and set λ=1.

That is, the expression diverges for $x\to y$, just like 1). The crucial point is that as the prefactor (x-y) decreases, the matrix element multiplying it diverges and faster.

All matrix elements of a commutator being available, as above, you may reconstitute your original operator equations from these, by insertion of resolutions of the identity on either side.

Cosmas Zachos
  • 62,595
  • Prof. Zachos, that $\langle a|a\rangle \neq 1 $ is a non-sequitur, because one can find very well find two operators satisfying the comm. relation in the Hilbert space, one of them being self-adjoint with normalizable eigenvectors. See section 12.2 of Brian Hall's book "Quantum Theory for Mathematicians". Then the rest of your argumentation becomes flawed as soon as you bring in the Dirac delta distribution. It follows that you initially worked in the distibutional space of the position eigenvectors in which you tried to insert the commutator which is not defined in that space at all. – DanielC Oct 18 '18 at 21:48
  • 1
    ? So you dismiss what every course following Dirac's book and his notation works out? Physicists were never stymied by unnormalizable states, or plane waves, or non-trace class operators. Fishermen on the pier hardly heed mathematical proofs of the non-existence of fish. Thanks to Dirac's distribution, physicists leapfrogged mathematicians by decades (... for a while). – Cosmas Zachos Oct 18 '18 at 21:54
  • 1
    I am sorry, but my respect for mathematically sound physics requires dismissing sources not in agreement with mathematics. Yes, QM is a mathematical theory and has been so for 91 years, with the first involvement of von Neuman at the suggestion of David Hilbert. Nothing can change that. One can choose to ignore mathematical rigor in physics, I choose not to. – DanielC Oct 18 '18 at 22:05
  • 1
    I, too, respect mathematical soundness: you can't fake nature. But basic properties of x and p are what most physicists work with, not manufactured oddball examples; unless someone finds a fly in the ointment in a specific relevant context. I gather the OP asked for guidance on a standard QM paradox--not a warning to "not go there". – Cosmas Zachos Oct 19 '18 at 00:05
  • 3
    @DanielC Are you saying anything using the Dirac delta function isn't valid? I just want to understand where you are coming from. – BioPhysicist Oct 19 '18 at 00:35
  • @AaronStevens it is valid only if test functions are used. Otherwise it is just hand-waving mathematics, as here: https://physics.stackexchange.com/questions/435513/weinbergs-lectures-on-quantum-mechanics-definition-of-momentum-operator or in the answer abovee – DanielC Oct 19 '18 at 16:06
  • 2
    @DanielC Cosmas recognizes this, but still discusses how to pull relevant conclusions. You don't need perfect mathematical rigor to understand the argument. Just like how in mathematics rigourous proofs are needed to make sure everything is valid, but we could still learn from just understanding the idea of the proof. In other words, mathematical rigor is very important, but I don't think it is always required when talking about or learning concepts, especially if the rigor would get in the way of a better learning experience. – BioPhysicist Oct 19 '18 at 17:15
  • 2
    There is hardly anything in the above that is not in Dirac's book--including the homogeneity of the delta function. Weyl, who introduced the fake paradox, also virtually resolved it with QM around the clock. – Cosmas Zachos Oct 19 '18 at 18:21